Chapin: Commentators have noted with concern the recent electoral success by extremist parties in several democratic...

jam0086@mix.wvu.edu on July 21, 2020

Question 4

Can someone please explain this question. Thank you

Reply
Create a free account to read and take part in forum discussions.

Already have an account? log in

shunhe on July 22, 2020

Hi @jam0086@mix.wvu.edu,

Thanks for the question! So let’s take a look at the stimulus. We’re told that extremist parties have had electoral success in several democratic countries. But, Chapin concludes, those successes don’t actually pose a threat to democracy in those countries. Why? Well, two reasons. The extremists have won pluralities, not majorities. And also, they only won when the more moderate parties were busy fighting among themselves.

Now we’re asked for something that allows the conclusion to be properly drawn if assumed; in other words, this is a strengthen with sufficient premise. And thinking about it a bit before we go into the answer choices, it’s likely going to have something to do with linking winning pluralities (not majorities) or moderate parties fighting among themselves or both to the idea of not posing a threat to democracy.

Now take a look at (A), which tells us that parties that win pluralities but not majorities never directly or indirectly effect changes in their country’s political arrangements. In real life, this probably isn’t true. But remember, we get to assume that this is true. And if it is true? The conclusion definitely follows. If (A) is true, then the extremist parties (which won pluralities but not majorities) can’t actually effect changes in political arrangements, directly or indirectly! And so they can’t be a threat to democracy in those countries. Since assuming (A), which is a super strong statement, lets us get to the conclusion, (A) is the correct answer.

Hope this helps! Feel free to ask any other questions that you might have.